dan
Thanks Received: 155
Atticus Finch
Atticus Finch
 
Posts: 202
Joined: March 10th, 2009
 
This post thanked 1 time.
 
 

Q21 - Doctor: Medication to reduce

by dan Wed Jul 28, 2010 5:26 pm

21. (C)
Question type: Assumption

The doctor states that one way of reducing blood pressure _ medication _ has unhealthy side effects, and so therefore other ways of reducing blood pressure _ exercising more and avoiding fatty foods _ are healthier. However, we’ve been told nothing about the negative side effects of these alternative methods. Therefore, (C) is the necessary assumption, and the correct answer.

(A) is not required for the argument to be true.
(B) relates to a different conclusion than the one in the argument.
(D) generalizes too far beyond the subject matter of the argument.
(E) is not required for the argument to be true, and generalizes beyond the scope of the argument.
 
kmewmewblue
Thanks Received: 1
Forum Guests
 
Posts: 57
Joined: April 18th, 2011
 
 
trophy
Most Thankful
 

Re: Q21 - Doctor: Medication to reduce

by kmewmewblue Thu Nov 24, 2011 2:19 am

Could anybody elaborate more on wrong answer choices?

(A) and (B) are still tempting...
User avatar
 
ohthatpatrick
Thanks Received: 3808
Atticus Finch
Atticus Finch
 
Posts: 4661
Joined: April 01st, 2011
 
This post thanked 2 times.
 
 

Re: Q21 - Doctor: Medication to reduce blood pressure

by ohthatpatrick Fri Nov 25, 2011 3:10 pm

I'd be happy to elaborate.

When we're doing Necessary Assumption, it's really helpful to keep in mind that they're asking us for the minimum that needs to be true to keep this argument from falling apart.

They're NOT asking us for a strong idea that would totally prove the conclusion (that's what Sufficient Assumption questions are asking for).

So TONS of Necessary Assumption answers can be easily identified as incorrect if we just look for terms that are too extreme or that go beyond the scope of the argument we were trying to make.


(A) is wrong because of the word "only". Pure and simple.

If I were arguing to someone "I know you want to go to a top law school, but you shouldn't go to Stanford law school. Yale or NYU is better."

Does that mean I'm assuming that "other than Stanford, the only other top law schools are Yale and NYU"?

Of course not. But that's what (A) is offering us. Just because the author has only discussed medication, exercise, and reducing fatty foods, that doesn't mean that those three things are the only ways to reduce blood pressure.

(B) is wrong because it has reversed the logic of the argument. If you want to represent the logic of someone's argument, you show it as
Premise --> Conclusion

To be tricky, the test may offer you the contrapositive of that (which is still a correct, identical idea):
~Conclusion --> ~Premise

The INCORRECT trap answers get this ordering wrong. They either give you:
Conclusion --> Premise (incorrect)
or
~Premise --> ~Conclusion (incorrect)

(B) is written as
Conclusion --> Premise

That ordering will ALWAYS be wrong.

(D) is wrong because of the extreme word "always". The author is only addressing one specific situation in which he believes that an alternative is healthier than the medication. He doesn't have to assume that's ALWAYS the case.

(E) doesn't match up to the facts of the argument. We're not choosing between medication vs. lifestyle changes on the basis that one is more effective than the other, while they have similar side effects.

Instead, we're choosing between medication vs. lifestyle changes on the basis that they have similar effectiveness, while one of them (supposedly, medication) has more side effects.

The "supposedly" part of that is why (C) is our correct answer. The author never addresses potential side effects of lifestyle changes.

Hope this helps.
 
sumukh09
Thanks Received: 139
Atticus Finch
Atticus Finch
 
Posts: 327
Joined: June 03rd, 2012
 
 
trophy
Most Thanked
trophy
First Responder
 

Re: Q21 - Doctor: Medication to reduce

by sumukh09 Sun Feb 24, 2013 3:30 am

Can B) be described as a premise booster?
User avatar
 
Crogati
Thanks Received: 2
Jackie Chiles
Jackie Chiles
 
Posts: 32
Joined: January 12th, 2013
 
 
 

Re: Q21 - Doctor: Medication to reduce

by Crogati Sun May 19, 2013 2:11 pm

I'm trying to think about this question on an even deeper level for practice:

I am wondering if choice D could be a sufficient assumption? It has much stronger language ("always") than the correct answer C, which is typical of sufficient assumptions, but unlike C, it doesn't mention side effects, only effectiveness.

Premise:
meds that reduce blood pressure have unhealthy side effects
+
Lifestyle changes reduce blood pressure just as effectively as meds do

Conclusion:
Healthier to rely on lifestyle changes than on meds to reduce blood pressure
Last edited by Crogati on Sun Jun 16, 2013 9:37 pm, edited 1 time in total.
User avatar
 
ohthatpatrick
Thanks Received: 3808
Atticus Finch
Atticus Finch
 
Posts: 4661
Joined: April 01st, 2011
 
 
 

Re: Q21 - Doctor: Medication to reduce

by ohthatpatrick Tue May 21, 2013 4:52 pm

In response to the question two posts back, (B) is not a premise booster.

(B) actually mentions the entire argument core, both premise and conclusion. The problem with (B) is that it shows the core in reverse order.

It shows
Conc --> Prem

But arguments function like
Prem --> Conc

In response to the last question, yes (D) works as a sufficient assumption, as long as we accept that "reducing blood pressure just as effectively" qualifies as "relieving a medical condition just as effectively".

I think LSAT would probably allow that, so (D) would qualify as sufficient, though not necessary.

Nice observation.

P.S. In order to qualify as a sufficient assumption, you don't HAVE to involve every premise idea.

For example:
Patrick teaches LSAT. Thus, he must be logical.

Surely we'd predict the Sufficient Assumption is "anyone who teaches LSAT must be logical".

But it would still be a correct Sufficient Assumption if it said "anyone whose name begins with 'P' is logical."

As long as a Sufficient Assumption succeeds in proving the conclusion, it's correct. It doesn't have to rope in all the ideas mentioned.
User avatar
 
uhdang
Thanks Received: 25
Atticus Finch
Atticus Finch
 
Posts: 227
Joined: March 05th, 2015
 
 
 

Re: Q21 - Doctor: Medication to reduce

by uhdang Wed Mar 11, 2015 11:44 pm

I've been told that in every argument there are infinite number of assumptions, and I am trying to identify at least some of them.. But for this question, other than the major assumption of "lifestyle change has less unhealthy impact on people than taking medication does", I can't think of any and I am just staring down at the question..

I think it is because they have narrowed down to comparing two methods other than opening out to lots of possibilities to be healthier by reducing blood pressure..

What would be assumptions (at least a couple of them) that we could make out from this argument? (those of which could develop into pre-phrasing answer choices?)

And for B),

Aside from mistaken reversal, what we are given from the passage about the effectiveness of lifestyle change (just as effective as taking medication) and what AC is giving us (lifestyle change is at least as effective as medication, meaning it could be more or equally effective) are different. So, in cases where we are given a different information, or contradicting information, than the passage, does that mean the answer choice "weakens" the premise?
"Fun"
User avatar
 
ohthatpatrick
Thanks Received: 3808
Atticus Finch
Atticus Finch
 
Posts: 4661
Joined: April 01st, 2011
 
This post thanked 1 time.
 
 

Re: Q21 - Doctor: Medication to reduce

by ohthatpatrick Fri Mar 13, 2015 1:56 pm

Contradicting a premise would indeed weaken the argument, but LSAT doesn’t ever contradict premises. They undermine a premise’s value / relevance / trustworthiness, but they don’t flat out contradict what was said.

Saying something slightly differently from a premise could weaken, strengthen, or do nothing to the argument. It depends.

This is an Assumption question, so seeing that an answer choice is distorting what was actually said is generally a red flag (if the distortion goes beyond what was said in the passage).

So … “at least as effectively” vs. “just as effective” … is definitely a noteworthy difference, and the answer choice goes beyond what we knew from the stimulus, making it wrong.

As far as the claim that there are infinite assumptions for any argument, that’s true but misleading.

Remember that in LR we are being tested on two completely different skills:
- Understanding the Mathematical Logic of an Argument
- Debating the truth value of claims by presenting evidence for/against

Another way to say this is that whenever we’re evaluating arguments, we’re looking for either
- Missing Logical Links
- Potential Objections / Alternative Explanations

The more you do LSAT, the more you’ll develop a feel for when something is testing a ‘black and white’ logic gap or a ‘shades of grey’ potential objection.

If I give you a short, symbolically repetitious argument like this:

Bob applied to Harvard. Thus, Bob applied to a good school.

There is just one missing logical link: “Harvard is a good school”

You want to think of Missing Logical Links as finite. There are normally only one or two in any argument.

But Potential Objections are infinite.

The Bob/Harvard argument only has one point of vulnerability: whether Harvard is a good school.

So if we wanted to spin off ‘infinite’ assumptions, we would just have to rule out potential objections that make it sound like Harvard ISN’T a good school.

- Harvard was NOT recently ranked as “America’s Worst School”
- Harvard students do not on average have poorer academic records than students from most other schools.
- At least some students receive a quality education from Harvard.
Etc, etc.

The same thing applies to this argument.

Since the scope of the conclusion is simply, “Which is healthier: X or Y”, the only logically relevant issues are good-health things about X and Y and bad-health things about X and Y.

We already know some good-health things: both Meds and Lifestyle are equally effective in reducing blood pressure.

We know a bad-health thing about Meds: meds have unhealthy side effects.

So in order to fight the author’s conclusion that Lifestyle is healthier, we’d have to hear bad-health things about Lifestyle/Exercise that sound worse than the healthy side effects of Meds.

(C) is pretty much the only way to sum the whole thing up succinctly: the bad-health stuff of Lifestyle is less bad than the bad-health stuff of Meds.

The only other way to generate possible answers here are to try to get more specific about the side effects, but achieve the same effect.

“The side effects of lifestyle changes are not significantly more likely to shorten one’s life span than those of medication”

“Relying on lifestyle changes to reduce blood pressure is possible without crucially endangering one’s life.”

So I think you should refine your notion of “infinite assumptions”. Again, true-ish but misleading. When I read this argument, I immediately think of (C) and expect it to be the correct answer, but that’s because this argument is really narrowed down.

Here’s an argument with what I would consider infinite assumptions:
Bob is in the other room crying. Thus, he must be cutting onions.

There is one, finite, logical link assumption “cutting onions can make someone cry”.

But there are infinite potential objection assumptions, such as “Bob is not watching a sad movie” / “Bob’s dog did not just die” / “Bob is not suffering an allergy attack right now”, etc.

Hope this helps.
User avatar
 
uhdang
Thanks Received: 25
Atticus Finch
Atticus Finch
 
Posts: 227
Joined: March 05th, 2015
 
 
 

Re: Q21 - Doctor: Medication to reduce

by uhdang Fri Mar 13, 2015 10:24 pm

Thanks for the post!
You've clarified so much for me!

So, ultimately, in Necessary assumption questions, for those that have "missing" link type, we could find finite number of "link" assumption and infinite amount of assumptions for "Potential Objection" just as you showed in Harvard example.

Then, for those questions that ask for "Potential Objection", could the argument NOT have the "link" assumption at all but just "potential objection" assumptions?

When I see "Potential Objection" necessary assumption questions, I just identified those "Potential objections" and finished up the analysis (when I couldn't easily identify a logical missing link). Should I have been looking for the link as well? or is it possible for an argument to be air-tight without the missing link (and LSAT is just picking on the argument by raising "potential objections")?
"Fun"
User avatar
 
ohthatpatrick
Thanks Received: 3808
Atticus Finch
Atticus Finch
 
Posts: 4661
Joined: April 01st, 2011
 
This post thanked 1 time.
 
 

Re: Q21 - Doctor: Medication to reduce

by ohthatpatrick Fri Mar 20, 2015 12:53 pm

I’m not sure I followed all your questions.

What do you mean by a “Potential Objection” Necessary Assumption question?

Are you referring to Flaw, which asks “how is this vulnerable to criticism”?

In any case, NONE of the arguments we will ever see in the Assumption Family (Flaw, Strengthen, Weaken, Nec/Suff Assumption, Principle-Strengthen) will ever be airtight as written.

Therefore, they always have missing links. But you can learn to have a good intuitive sense about whether the question you’re doing is testing “idea math” (i.e. logical links) or whether it’s testing debating skills (potential objections).

For instance, if I see this argument:

The lights are off. Therefore, no one is home.

It COULD be testing the missing link of “lights off -> no one home”, but I would assume that it’s really testing our ability to come up with alternative explanations for why the lights are off.

Whereas, if I see this argument:

Brazil refused to participate in the international G15 talks. Therefore, Brazil does not want to be part of the international community.

It COULD be testing the potential objection of “are there other times when Brazil HAS shown interest in the international community”, but it’s probably testing the missing link between “refused to participate in G15 -> doesn’t want to be part of int’l community”.

The reason I suspect Missing Link for the 2nd one is because there is symbol repetition going from Premise to Conclusion.
Prem: Brazil is X.
Conc: Brazil is Y.

The reason I suspect Potential Objection for the 1st example is because there is NOT symbol repetition and because the author IS trying to offer an explanation for a statistic/phenomenon.

You have to just stay flexible and potentially look out for both, but the more of these you do, the better you’ll get at having a strong sense of which one a given question is testing.
User avatar
 
uhdang
Thanks Received: 25
Atticus Finch
Atticus Finch
 
Posts: 227
Joined: March 05th, 2015
 
 
 

Re: Q21 - Doctor: Medication to reduce

by uhdang Fri Mar 20, 2015 7:31 pm

So, questions from "Assumption Family" will always have both at least one "missing link" and also at least one "potential objection." Got it!

Sorry about the "potential objection" issue. I did mean a possible flaw that an author could exploit using assumptions from the argument.

Thank you for the reply!
"Fun"
 
deedubbew
Thanks Received: 4
Forum Guests
 
Posts: 106
Joined: November 24th, 2013
 
 
 

Re: Q21 - Doctor: Medication to reduce

by deedubbew Sun Oct 11, 2015 2:13 am

What if there are problems other than side effects that effect the healthiness comparison between lifestyle changes and medication? I think the necessary assumption is that there are less unhealthy things related to lifestyle changes compared to medication. Not necessarily "side effects", just unhealthy things. It seems that answer choices that are narrower than the minimum sufficient assumption are okay if they are related to a premise and within the scope of the conclusion but not more narrow than the scope of conclusion if it is not related to a premise. ie. choice (A) which narrows the possibility of possible solutions to high blood pressure. Choice A is still within the scope of the conclusion, but not related to a premise. This also brings me to a question of, would it be okay to choose an answer choice with a scope that is more narrow than a premise? ie. "Some of the side effects of medication are more unhealthy than some of the side effects of lifestyle choices. "

You said “Relying on lifestyle changes to reduce blood pressure is possible without crucially endangering one’s life.” as an example that would work. But this sounds like a sufficient assumption. Am I wrong?

You mentioned the reversal of logic as an incorrect answer choice. Would the correct logic linking premise and concl be a correct answer? I know it would work in a sufficient condition question, but with what about nec assumption q’s? If the answer choice said, "if lifestyle changes and medication are equally effective, then lifestyle changes are healthier"; would that actually work with a nec assumption q? It seems to me like it would only work with sufficient assumption type questions.